If Negotiating is given at some time before Objections, then which one of the following must be true?

DavidW on February 26, 2020

Game Setup + Question 15

Hi! Could you please show the setup for this game, and how you would tackle question 15? Thanks so much :)

Replies
Create a free account to read and take part in forum discussions.

Already have an account? log in

Irina on February 27, 2020

@DavidW,

This is a linear game that requires us to determine the schedule of 3 short - G O P and 3 long - H N T seminars over three days. Each day exactly two seminars are given - 1 long and 1 short.

S __ __ __ G O P
L __ __ __ H N T
1 2 3

The rules tell us that:

(1) T will not be given until both G & O have been given.

Since G & O are both short seminars, we can infer that it is impossible for T to be given on day 1.
If T is given on day 2, it must come after G/O:

G/O G/O P
T

If T is given on day 3, it must either come after G/O OR it could come either before or after P.

G/O P G/O
T

P G/O G/O
T

G/O G/O P <- against rule (2)
T

(2) N will not be given until P has been given.
This rule tells us that N cannot be the first seminar give, and P cannot be the last seminar given.
It also tells us that the scenario above where T is the last seminar given on day 3 along with T is impossible because if P is on day 3, N must follow it on day 3.

If P(3) -> N (3) -> ~ T(3)

In other words, if P is given on day 3, T must be given on day 2 after G/O and N must be given on day 3 after P.


G/O G/O P
T N

The question asks us if N is given some time before O, which one of the following must be true?

In other words, we have the following chain P - N - O - T because P must come before N and T must come after O and G. We can, therefore, conclude that P must come sometime before O (C).

Let's look at the other answer choices:

(A) is incorrect because N could come after G:

P G O
H N T



(B) is incorrect because P could be given after G:

G P O
H N T

(D) is incorrect because H could be given after O:

P O G
N H T

(E) is incorrect because N could be given after H (see scenario for (B) above).

As long as P-N-O-T order is followed, it does not matter which seminars are given in between.

Let me know if this makes sense and if you have any other questions.

DavidW on March 1, 2020

Super helpful. Thank you!